On what intervals is the function shown in the graph increasing?


A) x<-4 and x > 2
B) x<-4 and -1 < x < 2
C) -4 2
D) -4

On What Intervals Is The Function Shown In The Graph Increasing?A) X&lt;-4 And X &gt; 2B) X&lt;-4 And

Answers

Answer 1

Answer:  [tex]-4 < x < -1[/tex] and [tex]x > 2[/tex]

Explanation:

The graph is increasing when the curve moves uphill as you move to the right. In other words, as x increases, y increases.

The first portion where the graph increases is from x = -4 to x = -1. In the span of -4 < x < -1 is when the graph goes uphill as shown by the red portion below. The blue portion is the second increasing piece that goes on forever upward, and this happens when x > 2.

On What Intervals Is The Function Shown In The Graph Increasing?A) X&lt;-4 And X &gt; 2B) X&lt;-4 And

Related Questions

plz help on a timed test will pay 10 points and make brainlest

What other countries have set up early earthquake warning systems? Which of these countries are in the Ring of Fire? Why is it important for these Ring of Fire countries to have warning systems?

Answers

Answer:

Mexico, Japan, South Korea, Taiwan, and the United States, Chile, Japan, the US west coast;  Chile, Japan, the US west coast; So all that is left for the people who live around the Ring of Fire is to be aware of the danger, perhaps to live further inland, build safer, earthquake-resistant housing, and for nations everywhere to improve oceanic and land-based early-warning systems to help minimize the risk to life

Step-by-step explanation:

e/kaid_274404147475337871026587/assig!
Khan Academy
Percent word problems
Of the 50 U.S. states, 4 have names that start with the letter W.
What percentage of U.S. states have names that start with the letter W?
0
Stuck? Review related articles/videos or use a hint.

Answers

Answer:

Step-by-step explanation:

4/50 = 0.08

0.08(100) = 8%

g A long-term study has revealed that a test for cancer in men is very effective. The study shows that 89% of the men for which the test is positive actually have cancer. If a man selected at random tests positive for cancer with this test, what is the probability that he does not have cancer

Answers

The answer is 89% because I had that Same question and it was 89%

geomtry plz help 15 points

Answers

Answer:

m∠O = 41°

Step-by-step explanation:

∠NOM=∠NMO=(4y-15)° (base angles of isos triangle)

7y+2(4y-15)=180 (angle sum of triangle)

  7y+8y-30=180

       15y-30=180

             15y=180+30

                  =210

                y=210÷15

                  =14

Hence, m∠O = (4y-15)°

                      = [4(14)-15]°

                      = (56-15)°

                      = 41°

what is 4% of 32? is it a percent, base, or amount

Answers

Answer:

1.28 - amount

Step-by-step explanation:

You would multiply 32 times 0.04 and get 1.28 which I assume would be an amount.

The amount of 4% of 32 is 1.28.

We have to determine what is 4% of 32.

What is the percentage?

This free percentage calculator computes a number of values involving percentages, including the percentage difference between two given values.

Suppose it is an x percent.

Therefore x multiply 32 times 0.04 and get 1.28 an amount.

To learn more about the percentage of visits:

https://brainly.com/question/24304697

#SPJ2

Note: Enter your answer and show all the steps that you use to solve this problem in the space provided.

You buy clothing at a sale. You buy a sweater at
7
10
of its original price of $35. Answer each question and show all your work.

a. How much money did you spend?
b. How much money did you save?
c. What fraction of the total original price did you save?

Answers

The amount that will be spent is $24.50, the amount saved will be $10.50, and the percentage saved will be 30%.

The amount that will be spent will be:

= 7/10 × $35 = $24.50

The amount that will be saved will be:

= $35 - $24.50 = $10.50

The fraction of the original price that is saved will be:

= 10.50/35 × 100

= 30%

Read related link on:

https://brainly.com/question/25763817

What is revolving credit?
A. Credit that provides a set amount of money for a specific purpose
B. Credit that is automatically renewed as debts are paid
C. Credit that requires users to pay a high interest rate
D. Credit that is issued by a financial institution other than a bank

Answers

Answer:

B. Credit that is automatically renewed as debts are paid

Hope this helps :)

Alf needs to borrow $15,000 to pay for his college tuition. He can borrow the money from his
parents at a rate of 3.55% interest compounded annually for 4 years, or he can borrow from his
local bank at a rate of 3.50% interest compounded continuously for 4 years
Manipulation of Numerical Data

Answers

Alf would prefer to borrow from his parents because their rates are more favourable.

The future value of the loan if he borrows from his parents is given by this formula: A(1 + r)^n

Where:

A = amount  

R = interest rate  

N = number of years  

$15,000(1 + 0.0355)^4 = $17,246.13

The future value of the loan if he borrows from the local bank is given by this formula: : A x e^r x N

Where:

A= amount e = 2.7182818 N = number of years r = interest rate

$15,000 x  2.7182818^0.035 x 4 = $62,137.18

To learn more about future value, please check: https://brainly.com/question/14640433

What’s the slope of 10,6 and 4, 1.2

Answers

let's firstly change the 1.2 to a fraction

[tex]1.\underline{2}\implies \cfrac{12}{1\underline{0}}\implies \cfrac{6}{5} \\\\[-0.35em] ~\dotfill\\\\ (\stackrel{x_1}{10}~,~\stackrel{y_1}{6})\qquad (\stackrel{x_2}{4}~,~\stackrel{y_2}{\frac{6}{5}}) \\\\\\ \stackrel{slope}{m}\implies \cfrac{\stackrel{rise} {\stackrel{y_2}{\frac{6}{5}}-\stackrel{y1}{6}}}{\underset{run} {\underset{x_2}{4}-\underset{x_1}{10}}}\implies \cfrac{~~ \frac{6-30}{5}~~}{-6}\implies \cfrac{~~ \frac{-24}{5}~~}{-6}\implies \cfrac{~~ -\frac{24}{5}~~}{-\frac{6}{1}}[/tex]

[tex]-\cfrac{\stackrel{4}{~~\begin{matrix} 24 \\[-0.7em]\cline{1-1}\\[-5pt]\end{matrix}~~}}{5}\cdot -\cfrac{1}{\underset{1}{~~\begin{matrix} 6 \\[-0.7em]\cline{1-1}\\[-5pt]\end{matrix}~~}}\implies \boxed{\cfrac{4}{5}}[/tex]

What is the coefficient of x in expression of 5mx​

Answers

Coefficient of this expression is 5 only.

- BRAINLIEST answerer

The width of a triangle is six more than twice the height. The area of the triangle is 88in2. Find the height and width of the triangle.

Answers

Answer:

w = 22 in, h = 8 in.

Step-by-step explanation:

Reading the problem, I immediately see two equations. This hints me towards a system of equations problem.

Statement 1 is The width of a triangle is six more than twice the height.

The width (w) of a triangle is (=) six (6) more (+) than twice (2) the height (h).

Equation 1: w = 6 + 2h

Statement 2 is The area of the triangle is 88 in2.

The formula for the area of a triangle is [tex]\frac{wh}{2}[/tex]. So, we get our second equation.

Equation 2: wh/2 = 88

We see that we have the same number of equations as unknowns, two. This is important as it lets us know that we can find two unknowns in the two equations. If there are more unknowns than equations, then we cannot find the values of all the variables. Look for another equation in the problem. However, if the # equations greater than or equal to number of unknowns, then we are ready to start solving!  So, we can now move to solving.

Equation 1 isolates one of the unknowns (w). So, we can replace w in the second equation with the right side of eq 1. Then, we simplify for h.

[tex]\frac{wh}{2} = 88[/tex]

[tex]\frac{(6 + 2h)h}{2} = 88[/tex]

[tex]\frac{6h + 2h^2}{2} = 88[/tex]

[tex]6h + 2h^2 = 176[/tex]

[tex]2h^2 + 6h - 176 = 0[/tex]

[tex]h = -11, 8[/tex] (assuming you know how to solve quadratics)

Since we know that heights of triangles cannot be negative, we find that [tex]h = 8[/tex] in.

For the final step, we plug [tex]h[/tex] into our first equation to find [tex]w[/tex].

[tex]w = 6 + 2h[/tex]

[tex]w = 6 + 2(8)[/tex]

[tex]w = 6 + 16[/tex]

[tex]w = 22[/tex]

So, the answer is w = 22 in, h = 8 in.

To check if we were right, we can plug the values into both equations to see if they are equal. I will skip equation 1 since we used that equation "as-is" to find w.

[tex]\frac{wh}{2} = 88[/tex]

[tex]\frac{22 * 8}{2} = 88[/tex]

[tex]11 * 8 = 88[/tex]

[tex]88 = 88[/tex]

The solution checks out!

I will give brainlist to the most helpful answer

Answers

Answer:

1. 4

2. 6

3. 10

4. 30

5. 48

6. 55

7. 68

8. 148

9. 102

10. 38

35. In her rough work, Janu added 2389 and 740 as shown here.
Is it okay to write the number 740 this way in the addition?
(Hint: You do NOT need to calculate)
(A) Yes, because 0 has no value so 74 is the same as 740.
(B) No, because the answer will be wrong if 0 is not written.
(C) No, because the digits 7 and 4 have NOT been arranged according to their value.
(D) Yes, because 7 is added with the hundred's digit and 4 with the ten's digit.

Answers

Answer:

D

Step-by-step explanation:

Juan wants to buy a video game for $63. He saves $12 every Friday. Part A. Create an equation to represent Juan's total savings, y, in dollars, after x Fridays. y​

Answers

Answer:

Answer in explanation.

Step-by-step explanation:

An equation to represent Juan's total savings y, in dollars, after x Fridays will be;

⇒ y = 12x

What is an expression?

Mathematical expression is defined as the collection of the numbers variables and functions by using operations like addition, subtraction, multiplication, and division.

Given that;

Juan wants to buy a video game for $63.

And, He saves $12 every Friday.

Now,

Let total dollars after x Friday = y

And, He saves $12 every Friday.

So, We can formulate;

⇒ Total money (y) = 12x

⇒ y = 12x

Thus, An equation to represent Juan's total savings y, in dollars, after x Fridays will be;

⇒ y = 12x

Learn more about the mathematical expression visit:

brainly.com/question/1859113

#SPJ2

Is 40 is 50% of 80 true or false
With step by step evidence

Answers

Answer:

please mark brainliest

it is true

Step-by-step explanation:

if you multiplie 80 by 0.50 you will get 40,

Evaluate each expression.
12
N
3
(2511)
-
DONE

Answers

Answer:

1/5

Step-by-step explanation:

[tex](25^{-3/2})^{1/3} = 25^{-3/2*1/3} = 25^{-1/2}=5^{-1/2*2}=5^{-1}=1/5[/tex]

Let's evaluate the expression,

→ (25^-3/2)^1/3

→ 25^-3/2×1/3

→ 25^-3/6

→ 5^-1/2×2

→ 5^-1 {or} 1/5

Hence, the answer is 1/5.

31 divided by 943.89

Answers

Answer:

0.032

Step-by-step explanation:

Answer:  0.03284281007

Step-by-step explanation:

La probabilidad de que julio salga con carla es 0.75 , y la probabilidad de que salga con marisol es de 0.50. Si la probabilidad de que salga con carla o marisol es 0.85; calcular la probabilidad de que salga con ambas a la vez

Answers

La probabilidad de que Julio salga con Marisol y Carla al mismo tiempo es 0.40

What is the addition rule of probability for two events?

For two events A and B, we have:

Probability that event A or B occurs = Probability that event A occurs + Probability that event B occurs - Probability that both the event A and B occur simultaneously.

This can be written symbolically as:

[tex]P(A \cup B) = P(A) + P(B) - P(A \cap B)[/tex]

Given that;

The probability that Julio goes out with Carla is 0.75, and

the probability that he goes out with Marisol is 0.50.

The probability that he goes out with Carla or Marisol is 0.85

To find:

The probability that he will date both at the same time.

Let we take:

A = event that Julio goes with Carla

B = event that Julio goes with Marisol, then, the given expression is symbolically represented as:

[tex]P(A) = 0.75\\[/tex][tex]P(B) = 0.50[/tex][tex]P(A \cup B) = 0.85[/tex]

To find: [tex]P(A \cap B)\\[/tex]

Using the addition rule, we get:

[tex]P(A \cup B) = P(A) + P(B) - P(A \cap B)\\0.85 = 0.75 + 0.50 - P(A \cap B)\\P(A \cap B) = 1.25 - 0.85 = 0.40[/tex]

Thus, the probability that Julio will date Marisol and Carla at the same time is 0.40

Learn more about probability here:

brainly.com/question/1210781

The probability that Julio will date Marisol and Carla at the same time is 0.40.

Here, we have, For two events A and B, we have:

Probability that event A or B occurs = Probability that event A occurs + Probability that event B occurs - Probability that both the event A and B occur simultaneously.

This can be written symbolically as:

P(A ∪ B) = P(A) + P(B) - P(A ∩ B)

Given that;

The probability that Julio goes out with Carla is 0.75, and

the probability that he goes out with Marisol is 0.50.

The probability that he goes out with Carla or Marisol is 0.85

To find:

The probability that he will date both at the same time.

Let we take:

A = event that Julio goes with Carla

B = event that Julio goes with Marisol, then, the given expression is symbolically represented as:

P(A) = 0.75

P(B) = 0.50

P(A∪B)= 0.85

To find: P(A ∩ B)

Using the addition rule, we get:

P(A ∪ B) = P(A) + P(B) - P(A ∩ B)

0.85 = 0.75 + 0.50 - P(A ∩ B)

P(A ∩ B) = 0.40

Thus, the probability that Julio will date Marisol and Carla at the same time is 0.40.

Learn more about probability here:

brainly.com/question/1210781

#SPJ3

complete question:

The probability that Julio dates Carla is 0.75, and the probability that he dates Marisol is 0.50. If the probability that he goes out with carla or marisol is 0.85; Calculate the probability that he will date both at the same time.

How many pounds are in 1 1/2 pounds and 8 ounces? There are _____ pounds in 1 1/2 pounds and 8 ounces.

Answers

Answer:

Hope this helps

Step-by-step explanation:

Since 16 ounces equal 1 pound, we’ll add 16 + 16, which is 32 ounces. For one half of a pound, simply divide the amount of ounces in a pound (16) by half. 16 divided by 2 is 8, so in total your answer would be 40 ounces in 2 1/2 pounds.

please help this is for my study guide thanks! ​

Answers

The volume is 29.32 <3

Write the equation of the function graphed below:

a. f(x) = 2x−−√3 –4
b. f(x) = 2x−−√3 + 4
c. f(x) =x−−√3 + 4
d. f(x) =x−−√3 – 4

Answers

Answer:

I have a couple questions before I answer, what do the 2 dashes represent and are the 4s also under the square root?


A high school track team's long jump record is 22 ft 6 3/4in. This year, Arthur's best long jump is 21 ft 10 1/2in. If long jumps are measured to the nearest quarter inch, how much farther must
Arthur jump to break the record?
To beat the school long jump record,

Arthur must jump an additional ___inches.

(Type a whole number, fraction, or mixed number.)

Answers

Arthur must jump an additional 8.25 inches so as to break the record.

1 foot =  12 inches

Long jump record = 22 ft 6 3/4in = 22 ft 6.75 in = (22ft * 12 in per ft) + 6.75 in

Long jump record = 270.75 in

Arthur's long jump record = 21 ft 10 1/2in = 21 ft 10.5 in = (21ft * 12 in per ft) + 10.5 in

Arthur's long jump record = 262.5 ft.

To break the record, the distance needed to be jump by Arthur = 270.75 in - 262.5 ft. = 8.25 in

Arthur must jump an additional 8.25 inches so as to break the record.

Find out more at: https://brainly.com/question/20796404

geomtry plzzz help 15 points

Answers

Answer:

19

Step-by-step explanation:

they give you xz, and they want half, so just divide 38 by 2

Solve the equation: 3.017 + k = 5.134

Answers

Answer:

k=2.117

Step-by-step explanation:

Answer:

3.017 + k = 5.134

So, k = 5.134 - 3.017

So, k = 2.117

math
solve this pls lol

Answers

Answer:

The two triangles are related by congruent angles/similarity, so the triangles are similar.

Have an awesome day! :D

which number will reach if we move 6 steps to the right -3

Answers

Answer:

If you meant -3 + 6, then the correct answer is 3.

You should put some extra effort in composing questions with all the required information, like a picture maybe.

Answer:

your answer has to be 3

Step-by-step explanation:

if you were to move to the right in number line you have to add so -3+6 is equals to 3

how to find the volume of an oven with the inside dimensions 3 ft by 1.5 ft by 3 ft

Answers

9514 1404 393

Answer:

  13.5 ft³

Step-by-step explanation:

The volume of a cuboid is the product of its length, width, and height:

  V = (3 ft)(1.5 ft)(3 ft) = 13.5 ft³

The volume of the oven is 13.5 ft³.

__

You may see this as the formula ...

  V = LWH

Suppose a jar contains 17 red marbles and 32 blue marbles. If you reach in the jar and pull out 2 marbles at random, find the probability that both are red.

Answers

There are 49 total marbles, the chance of you getting a red is 17/49, and if you don’t replace the marble and pull out another one, the probability of that being red is 17/49*16/48=17/147. That’s approximately a .12% chance.
16/48=1/3

The probability that both are red marbles = [tex]\bold{\frac{17}{147} }[/tex]

What is probability?

"It is finding out the possibilities of the occurrence of an event."

Formula to find the probability of an event:

"P(A) = n(A) / n(S)

where, n(A) is the number of favorable outcomes of an event A

n(S) is the total number of outcomes for an experiment"

For given example,

A jar contains 17 red marbles and 32 blue marbles.

If we pull out 2 marbles at random, we need to find the probability that both are red.

The number of possible outcomes,

[tex]\Rightarrow n(S)=^{49}C_2\\\\ \Rightarrow n(S)=\frac{49!}{2!(49-2)!}\\\\ \Rightarrow n(S)=1176[/tex]

The number of possible outcomes of selecting both the red marbles.

[tex]\Rightarrow n(A)=^{17}C_2\\\\ \Rightarrow n(A)=\frac{17!}{2!(17-2)!}\\\\ \Rightarrow n(A)=136[/tex]

The probability that both the marbles are red,

[tex]\Rightarrow P(A)=\frac{n(A)}{n(S)}\\\\ \Rightarrow P(A)=\frac{136}{1176}\\\\ \Rightarrow P(A)=\frac{17}{147}[/tex]

Therefore, the probability that both are red marbles = [tex]\bold{\frac{17}{147} }[/tex]

Learn more about the probability here:

brainly.com/question/11234923

#SPJ2

Evaluate this function: h(t) = |t+2| + 3; Find h(6)

Answers

Answer:

Step-by-step explanation:

h(6) = |6 + 2| + 3 = |8| + 3 = 8 + 3 = 11

consider the following matrix. I need to know if the system has zero or infinite solutions or just one solution and if the matrix does/does not have an inverse.​

Answers

Answer:

I think it would be no solution

Step-by-step explanation:

since there is the same slope, the line would never cross. Also the determinate is 0 or 1/0, so there would be no solution

Other Questions
Which function represents a translation of the graph of v=x^2by 10 units to the left?O A. v=x^2+10O B. v=(x+10)^2O C. y=10x^2O D, y=(x-10)^2 What is the slope of the line y=5-4x ?Show your work Please take a look at the picture 10. What is Newton's 3rd Law?O A. For every action there is an equal and opposite reaction.B. Acceleration depends on two variables, the mass of the object and the amount offorce.C. An object at rest will stay at rest, an object in motion will stay in motion, unless anunbalanced force acts upon it.D. The amount of matter in an object.7. What is Newton's 2nd Law? * f(x)=2x^2 + x - 6/ x-1 Which of the following adjectives most closely describes the tone of the passage below (paragraph 4)?About me lie the remnants of Solo. My equipment is properly secured, vital systems are functioning, and daily priorities are set, priorities not to be argued with. I somehow rise above mutinous apprehension, fear, and pain. I am captain of my tiny ship in treacherous waters. I escaped the confused turmoil following Solos loss, and I have finally gotten food and water. I have overcome almost certain death. I now have a choice: to pilot myself to a new life or to give up and watch myself die. I choose to kick as long as I can.Answer choices for the above questionA. determinedB. apprehensiveC. defeatedD. excited logb(x-1) + logb(x+2) = logb(8-2x) A machine depreciates by 1/4 of its value each year. If it cost $50,000 new,what is its value after 8 years? What type of promise to pay does a promissory note represent? In a school there are 200 boys and 800 girls. Find the percentage of girls in that school. What is the difference between an f donor and an hfr donor?. 1 234which one is it What European countries still own territory in Latn America? Which postulate or theorem proves that these two triangles arecongruent?O ASA Congruence PostulateO AAS Congruence TheoremO HL Congruence TheoremO SAS Congruence Postulateo An atom has 5 protons, 4 electrons, and 5 neutrons. Whichof the following reasons represents why this is an atom ofthe element boron?A ) The atom has 5 neutrons, just like every atom of boronB ) The atom has 4 electrons, just like every atom of boronC ) The atom has 14 total particles, in a combination of 5-5-4 just like every atom of boron D ) The atom has 5 protons, just like every atom of boron< PLEASE HELP ASAP! Ivan put 500 000ISK (Icelandic Krona) and Blake put 700 000ISK into savings accountsat different banks.After 8 years, Ivan and Blake have the same amount of money in their accounts.They both receive interest compounded quarterly and Blake has an annual interest ratethat is half of Ivan's annual interest rate.Work out the annual interest rate that Blake receives. PLEASE HELP POR FAVOR SOMEONE IM DESPERATE tHIS IS DUE IN THE mORiNG! 7. A bus covers a certain distance in 60 minutes if it runs at a speed of 60 km/hr.What must be the speed of the bus in order to reduce the time of journey by 40minutes? Help please. I will be happy!!!! first permanent english settlement to the new world?